Calculating [L_i,L_j] for Commutator Relation: A Step-by-Step Guide

I have simplified it till it's a sum over m and k only. But that doesn't reduce to 2...Yes it does. You are summing over two index values. Each index has 3 values. 3*3 = 9Yes it does. You are summing over two index values. Each index has 3 values. 3*3 = 9Oh I just realised, the solution I got is for ##[L_i,L_l]## not ##[L_i,L_j]##. Right, but you already realize how to get the solution for ##[L_i,L_j]##. It's almost there.[L_i,L_j] = \
  • #1
unscientific
1,734
13

Homework Statement



Find ##[L_i,L_j]##.

Homework Equations



[tex][x_i,p_j] = \delta_{ij}i\hbar[/tex]

The Attempt at a Solution



[tex][L_i,L_j] = \epsilon_{ijk}\epsilon_{jlm} [x_jp_k,x_lp_m][/tex]
[tex] = \left( \delta_{jl}\delta_{km} - \delta_{jm}\delta_{kl}\right)[x_jp_k,x_lp_m][/tex]
[tex] = [x_jp_k,x_jp_k] - [x_jp_k,p_kx_j][/tex]
[tex] = p_kx_jx_jp_k - x_jp_kp_kx_j[/tex]

Since ##x_jp_k = p_kx_j## for j≠k, we swap the two in the second term:

[tex] = p_kx_jx_jp_k - x_jp_kx_jp_k[/tex]
[tex] = [p_k,x_j]x_jp_k[/tex]
[tex] = -i\hbar \delta_{jk}x_jp_k[/tex]

I think the answer is something like ##i\hbar L_k##.
 
Physics news on Phys.org
  • #2
unscientific said:
[tex][L_i,L_j] = \epsilon_{ijk}\epsilon_{jlm} [x_jp_k,x_lp_m][/tex]

You've used j as a dummy summation index on the right, which leads to trouble because on the left j is a fixed index. You'll need to use a dummy index other than j when you express ##L_i##.
 
  • #3
TSny said:
You've used j as a dummy summation index on the right, which leads to trouble because on the left j is a fixed index. You'll need to use a dummy index other than j when you express ##L_i##.

[tex][L_i, L_l] = \epsilon_{ijk}\epsilon_{lmn}[x_jp_k,x_mp_n][/tex]
[tex] = (\delta_{jm}\delta_{kn} - \delta_{jn}\delta_{km})[x_jp_k, x_mp_n][/tex]
[tex] = -[x_jp_k, x_kp_j] [/tex]
[tex] = -\left( [x_jp_k,x_k]p_j + x_k[x_jp_k,p_j]\right) [/tex]
[tex] = -\left( x_j[p_kx_k]p_j + x_k[x_j,p_j]p_k\right)[/tex]
[tex] = i\hbar \left( x_jp_j - x_kp_k\right)[/tex]
[tex] = 0[/tex]
 
Last edited:
  • #4
unscientific said:
[tex][L_i, L_l] = \epsilon_{ijk}\epsilon_{lmn}[x_jp_k,x_mp_n][/tex]
[tex] = (\delta_{jm}\delta_{kn} - \delta_{jn}\delta_{km})[x_jp_k, x_mp_n][/tex]

[tex]\epsilon_{ijk}\epsilon_{lmn} \neq (\delta_{jm}\delta_{kn} - \delta_{jn}\delta_{km})[/tex]
(Note that indices ##i## and ##l## appear on the left but not on the right.)
 
  • #5
TSny said:
[tex]\epsilon_{ijk}\epsilon_{lmn} \neq (\delta_{jm}\delta_{kn} - \delta_{jn}\delta_{km})[/tex]
(Note that indices ##i## and ##l## appear on the left but not on the right.)

Ah that's right, for that identity to work they must have the same leftmost letter.
 
  • #6
Yes. Then you would be summing over the leftmost index of each Levi-Civita symbol. But, in your case there is no summation (yet).
 
  • #7
TSny said:
Yes. Then you would be summing over the leftmost index of each Levi-Civita symbol. But, in your case there is no summation (yet).

Ok I've given it another go!

[tex][x_jp_k, x_mp_n] = [x_jp_k, x_m]p_n + x_m[x_jp_k, p_n][/tex]
[tex] = \left( x_j[p_k, x_m]p_n + x_m[x_j, p_n]p_k\right) [/tex]
[tex] = -i\hbar \left( \delta_{km}x_jp_n - \delta_{jn}x_mp_k\right] [/tex]

Now applying ##\epsilon_{ijk}\epsilon_{lmn}##:

[tex] = -i\hbar \left( \epsilon_{ijk}\epsilon_{lkn}x_jp_k - \epsilon_{ijk}\epsilon_{lmj}x_mp_k\right)[/tex]

[tex] = -i\hbar \left( -\epsilon_{kij}\epsilon_{kln}x_jp_n + \epsilon_{jik}\epsilon_{jlm}x_mp_k\right)[/tex]

[tex] = i\hbar \left[ (\delta_{il}\delta_{jn} - \delta_{in}\delta_{jl})x_jp_n - (\delta_{il}\delta_{km} - \delta_{im}\delta_{kl})x_mp_k\right][/tex]

Now, ##i=m=n## and ##j=k=l##:

[tex] = i\hbar \left( x_jp_j - x_jp_i -x_jp_j+x_ip_j\right) [/tex]

[tex] = i\hbar (x_ip_j - x_jp_i)[/tex]

[tex][L_i,L_j] = i\hbar \epsilon_{kij}x_ip_j[/tex]

[tex] [L_i,L_j] = i\hbar L_k[/tex]
 
  • #8
unscientific said:
Ok I've given it another go!

[tex][x_jp_k, x_mp_n] = [x_jp_k, x_m]p_n + x_m[x_jp_k, p_n][/tex]
[tex] = \left( x_j[p_k, x_m]p_n + x_m[x_j, p_n]p_k\right) [/tex]
[tex] = -i\hbar \left( \delta_{km}x_jp_n - \delta_{jn}x_mp_k\right] [/tex]

Now applying ##\epsilon_{ijk}\epsilon_{lmn}##:

[tex] = -i\hbar \left( \epsilon_{ijk}\epsilon_{lkn}x_jp_k - \epsilon_{ijk}\epsilon_{lmj}x_mp_k\right)[/tex]

[tex] = -i\hbar \left( -\epsilon_{kij}\epsilon_{kln}x_jp_n + \epsilon_{jik}\epsilon_{jlm}x_mp_k\right)[/tex]

[tex] = i\hbar \left[ (\delta_{il}\delta_{jn} - \delta_{in}\delta_{jl})x_jp_n - (\delta_{il}\delta_{km} - \delta_{im}\delta_{kl})x_mp_k\right][/tex]

OK. Looks good.

Now, ##i=m=n## and ##j=k=l##:

[tex] = i\hbar \left( x_jp_j - x_jp_i -x_jp_j+x_ip_j\right) [/tex]

From your post #3 you are evaluating ##[L_i, L_l]##. So the indices ##i## and ##l## are fixed.
Have another go at simplifying [tex] i\hbar \left[ (\delta_{il}\delta_{jn} - \delta_{in}\delta_{jl})x_jp_n - (\delta_{il}\delta_{km} - \delta_{im}\delta_{kl})x_mp_k\right][/tex]
 
  • #9
TSny said:
Have another go at simplifying [tex] i\hbar \left[ (\delta_{il}\delta_{jn} - \delta_{in}\delta_{jl})x_jp_n - (\delta_{il}\delta_{km} - \delta_{im}\delta_{kl})x_mp_k\right][/tex]

[tex] i\hbar \left[ (\delta_{il}\delta_{jn} - \delta_{in}\delta_{jl})x_jp_n - (\delta_{il}\delta_{km} - \delta_{im}\delta_{kl})x_mp_k\right][/tex]

[tex] = i\hbar [\delta_{im}\delta_{kl}x_mp_k - \delta_{in}\delta_{jl}x_jp_n][/tex]

[tex] = i\hbar [(\delta_{im}\delta_{kl} - \delta_{ik}\delta_{ml})x_mp_k][/tex]
 
  • #10
OK. You can keep going and simplify further. Note that you are summing over m, k, [STRIKE]j[/STRIKE], and [STRIKE]n[/STRIKE].

[EDIT: Sorry. I see you've reduced it to just summing over m and k]
 
Last edited:
  • #11
TSny said:
OK. You can keep going and simplify further. Note that you are summing over m, k, j, and n.

I have simplified it till it's a sum over m and k only. But that doesn't reduce to 2 epsilons
 
  • #12
OK. What is your final expression after summing over m and k?
 
Last edited:
  • #13
the deltas don't seem to combine to give an epsilon
 
  • #14
We'll see how to get an epsilon factor later. For now, what expression do you get after summing over m and k?
 
  • #15
TSny said:
We'll see how to get an epsilon factor later. For now, what expression do you get after summing over m and k?

Actually, check out post #7. I solved it already by expanding. I thought you meant that there was a way of making use of the identity ##(\epsilon_{1}\epsilon_{2} = \delta\delta - \delta\delta)## to get the epsilon more explicitly..

In post #7, the epsilon was drawn out by observation of a cross product ##x_ip_j - p_ix_j##
 
  • #16
Yes, in post 7 you are getting the right type of result. It's confusing, because you started out post 7 as a continuation of post 3 where you were calculating ##[L_i, L_l]##, but somehow ended up with an expression for ##[L_i, L_j]##.

But if you now feel that you understand all the steps to get from ##[L_i, L_j]## to ## i \hbar (x_i p_j - p_i x_j )##, you are essentially done.

If you let ##k## denote the direction perpendicular to the ##i##-##j ## plane,then you have shown that ##[L_i, L_j] = i \hbar (x_i p_j - p_i x_j) = i \hbar L_k##

Can you show that this is equivalent to ##[L_i, L_j] = i \hbar \epsilon_{ijm} L_m## with summation over ##m##?
 
  • #17
TSny said:
Yes, in post 7 you are getting the right type of result. It's confusing, because you started out post 7 as a continuation of post 3 where you were calculating ##[L_i, L_l]##, but somehow ended up with an expression for ##[L_i, L_j]##.

But if you now feel that you understand all the steps to get from ##[L_i, L_j]## to ## i \hbar (x_i p_j - p_i x_j )##, you are essentially done.

If you let ##k## denote the direction perpendicular to the ##i##-##j ## plane,then you have shown that ##[L_i, L_j] = i \hbar (x_i p_j - p_i x_j) = i \hbar L_k##

Can you show that this is equivalent to ##[L_i, L_j] = i \hbar \epsilon_{ijm} L_m## with summation over ##m##?

It's ##[L_i, L_l] = i\hbar (x_ip_l - p_ix_l) = i\hbar L_m## I'm still missing an ##\epsilon## though..
 
Last edited:
  • #18
OK, that's correct with the understanding that ##m## refers to the index that defines the direction perpendicular to the ##i##-##l## plane and that the indices ##i, l## and ##m## are in cyclic order.

The nice thing is that the one expression [tex][L_i, L_j] = i \hbar \sum_{k=1}^{3} \epsilon_{ijk}L_k = i \hbar \epsilon_{ijk}L_k [/tex] automatically takes care of everything. In the last expression the summation over ##k## is assumed (using the Einstein summation convention for repeated indices).
 
  • #19
TSny said:
OK, that's correct with the understanding that ##m## refers to the index that defines the direction perpendicular to the ##i##-##l## plane and that the indices ##i, l## and ##m## are in cyclic order.

The nice thing is that the one expression [tex][L_i, L_j] = i \hbar \sum_{k=1}^{3} \epsilon_{ijk}L_k = i \hbar \epsilon_{ijk}L_k [/tex] automatically takes care of everything. In the last expression the summation over ##k## is assumed (using the Einstein summation convention for repeated indices).

I seem to be missing an ##\epsilon## in my final answer..
 
  • #20
We have ##[L_i, L_j] = i \hbar (x_ip_j - x_jp_i)##.

The right hand side contains ##(x_ip_j - x_jp_i)##, which is the component of angular momentum that is perpendicular to the ##i##-##j## plane. A convenient way to express this component of angular momentum is ##\epsilon_{ijk}L_k## (where summation over the index ##k## is assumed). So the epsilon is "put in by hand" as a way of expressing the angular momentum component that is perpendicular to the ##i##-##j## plane.

Thus, we can write

##[L_i, L_j] = i \hbar \epsilon_{ijk}L_k##

As an example, if ##i = 1## and ##j = 2##, then ##[L_1, L_2] = i \hbar \epsilon_{12k}L_k = i \hbar L_3##.
 
  • #21
TSny said:
We have ##[L_i, L_j] = i \hbar (x_ip_j - x_jp_i)##.

The right hand side contains ##(x_ip_j - x_jp_i)##, which is the component of angular momentum that is perpendicular to the ##i##-##j## plane. A convenient way to express this component of angular momentum is ##\epsilon_{ijk}L_k## (where summation over the index ##k## is assumed). So the epsilon is "put in by hand" as a way of expressing the angular momentum component that is perpendicular to the ##i##-##j## plane.

Yeah, but matematically ##(x_ip_j - x_jp_i)## simply means ##\vec{x}##x##\vec{p}##. So by putting an epsilon sign, it becomes ##\epsilon_{ijk}x_jp_k## which is simply ##L_k##. By putting another epsilon, won't you get an extra factor of epsilon?
 
  • #22
unscientific said:
Yeah, but matematically ##(x_ip_j - x_jp_i)## simply means ##\vec{x}##x##\vec{p}##.

##(x_ip_j - x_jp_i) = (\vec{x} \times \vec{p})_k## where ##k## refers to the component of the cross product that is perpendicular to the i-j plane.

So by putting an epsilon sign, it becomes ##\epsilon_{ijk}x_jp_k## which is simply ##L_k##. By putting another epsilon, won't you get an extra factor of epsilon?

I'm not following. ##\epsilon_{ijk}x_jp_k## represents the ##i^{th}## component of the cross product ## \vec{x} \times \vec{p}##. (Note the repeated indices j and k in your expression, which means you are summing over j and k.)

I don't see how you would get two epsilon factors. For fixed i and j, ##(x_ip_j - x_jp_i) = \epsilon_{ijk}L_k## with just one epsilon factor (and summation over the index k).

In summary, you have for fixed i and j, [tex][L_i, L_j] = i \hbar (x_ip_j - x_jp_i) = i\hbar \epsilon_{ijk}L_k[/tex]
 
  • #23
TSny said:
##(x_ip_j - x_jp_i) = (\vec{x} \times \vec{p})_k## where ##k## refers to the component of the cross product that is perpendicular to the i-j plane.
I'm not following. ##\epsilon_{ijk}x_jp_k## represents the ##i^{th}## component of the cross product ## \vec{x} \times \vec{p}##. (Note the repeated indices j and k in your expression, which means you are summing over j and k.)

I don't see how you would get two epsilon factors. For fixed i and j, ##(x_ip_j - x_jp_i) = \epsilon_{ijk}L_k## with just one epsilon factor (and summation over the index k).

In summary, you have for fixed i and j, [tex][L_i, L_j] = i \hbar (x_ip_j - x_jp_i) = i\hbar \epsilon_{ijk}L_k[/tex]

Because ##L_k## is defined as ##\epsilon_{kij}x_ip_j ##
 
Last edited:
  • #24
any input?
 
  • #25
unscientific said:
any input?
About what? Are you still talking about this:

unscientific said:
It's ##[L_i, L_l] = i\hbar (x_ip_l - p_ix_l) = i\hbar L_m## I'm still missing an ##\epsilon## though..
You want to find an equality that holds for all ##i,l\in\{1,2,3\}## such that ##i\neq l##. If m=1, then ##[L_i,L_l]=i\hbar L_m## can't possibly hold for all ##i,l\in\{1,2,3\}## such that ##i\neq l##. Same thing if m=2, or if m=3.

The right-hand side (and every intermediate step) must depend on i and j, and nothing else.

unscientific said:
Because ##L_k## is defined as ##\epsilon_{kij}x_ip_j ##
Yes, we have ##L_k=\epsilon_{kij} x_i p_j##. Now what does that imply about ##\epsilon_{ijk}L_k##?

unscientific said:
Yeah, but matematically ##(x_ip_j - x_jp_i)## simply means ##\vec{x}##x##\vec{p}##.
Depending on the values of i and j, ##x_ip_j - x_jp_i## is either equal to 0, or to one of the components of ##\vec x\times\vec p##. Which component that is depends on the values of i and j. If i≠j, then it's the kth component, where k is the unique element of {1,2,3} such that ##\epsilon_{ijk}\neq 0##.
 
Last edited:
  • Like
Likes 1 person
  • #26
Fredrik said:
About what? Are you still talking about this:


You want to find an equality that holds for all ##i,l\in\{1,2,3\}## such that ##i\neq l##. If m=1, then ##[L_i,L_l]=i\hbar L_m## can't possibly hold for all ##i,l\in\{1,2,3\}## such that ##i\neq l##. Same thing if m=2, or if m=3.

The right-hand side (and every intermediate step) must depend on i and j, and nothing else.


Yes, we have ##L_k=\epsilon_{kij} x_i p_j##. Now what does that imply about ##\epsilon_{ijk}L_k##?


Depending on the values of i and j, ##x_ip_j - x_jp_i## is either equal to 0, or to one of the components of ##\vec x\times\vec p##. Which component that is depends on the values of i and j. If i≠j, then it's the kth component, where k is the unique element of {1,2,3} such that ##\epsilon_{ijk}\neq 0##.

To summarize:
[tex][L_i,L_j] = i\hbar \epsilon_{kij}L_k[/tex]
 
  • #27
Right, that last paragraph in my post is saying that ##x_ip_j-x_jp_i=\varepsilon_{ijk}L_k##. Note that there's a sum over k, so there are three terms on the right-hand side, but if ##i\neq j##, then two of them are zero because in those two k is equal to either i or j. And if ##i=j##, they're all zero, which is great, because then the left-hand side is zero too.

If you prefer a calculation over a wordy argument like this, you just calculate ##\varepsilon_{ijk}L_k## explicitly:
$$\varepsilon_{ijk}L_k=\varepsilon_{ijk}\varepsilon_{kmn}x_mx_n=\cdots.$$
Is everything crystal clear now?
 
Last edited:
  • Like
Likes 1 person

1. What is a commutator relation?

A commutator relation is a mathematical expression that describes the relationship between two operators. Specifically, in quantum mechanics, the commutator relation [L_i, L_j] is used to describe the relationship between two angular momentum operators, L_i and L_j.

2. How is a commutator relation calculated?

The commutator relation [L_i, L_j] is calculated by taking the difference between the two operators multiplied in two different orders. Mathematically, it can be expressed as [L_i, L_j] = L_iL_j - L_jL_i.

3. What does a non-zero commutator relation indicate?

A non-zero commutator relation between two operators indicates that the operators do not commute, meaning that the order in which they are applied matters. In the case of [L_i, L_j], this means that the angular momentum operators L_i and L_j do not commute with each other.

4. What is the physical significance of a commutator relation?

The commutator relation [L_i, L_j] has physical significance in quantum mechanics as it is related to the uncertainty principle. A non-zero commutator relation indicates that the two operators do not have simultaneously well-defined values, and therefore, their measurements are subject to uncertainty.

5. Can commutator relations be extended to other operators besides angular momentum?

Yes, commutator relations can be extended to other operators besides angular momentum. In quantum mechanics, commutator relations are used to describe the relationships between various operators, such as position and momentum operators, energy and time operators, and more.

Similar threads

  • Advanced Physics Homework Help
Replies
2
Views
4K
  • Advanced Physics Homework Help
Replies
4
Views
7K
  • Advanced Physics Homework Help
Replies
2
Views
5K
  • Advanced Physics Homework Help
Replies
1
Views
3K
  • Advanced Physics Homework Help
Replies
2
Views
4K
  • Advanced Physics Homework Help
Replies
3
Views
2K
  • Advanced Physics Homework Help
Replies
7
Views
1K
Replies
4
Views
3K
Replies
10
Views
2K
Replies
5
Views
25K
Back
Top